0 Daumen
249 Aufrufe

\( \sum\limits_{k=1}^{\infty}{\frac{4k+4}{k^2*(k+2)^1/2 }} \) → also k^2*(k+2)^(1/2)


Ich habe es mit dem Majoritätskriterium versucht, scheitere aber immer, wenn ich |bk| suche.

Es heißt ja: |ak| ≤ |bk|

Mein Ansatz:

(4k+4)/(k^2*(k+2)^(1/2)) im Betrag


aber ab hier weiß ich nicht, wie ich weiter abschätzen soll.

Avatar von

1 Antwort

0 Daumen

Hallo

im Nenner k+2 durch k ersetzen Nenner verkleinert vergrößert den Bruch, im Zähler 4 durch 4k vergrößern, dann hast du 8k/k^2.5 also 8/k^1.5

das ist grob aber führt zum Ziel

Gruß lul

Avatar von 107 k 🚀

Kann es sein, dass die Reihe divergent ist?

\(\sum\limits_{k=1}^{\infty}{\dfrac{a}{k^r}}\)  mit  a,r konstant

ist für r >1 konvergent

Vielen Dank!

Ein anderes Problem?

Stell deine Frage

Willkommen bei der Mathelounge! Stell deine Frage einfach und kostenlos

x
Made by a lovely community